Proving that Supremum of (a,b) is Equal to b

  • Thread starter jgens
  • Start date
  • Tags
    Supremum
In summary: The proof seems fine to me.Also, it's pretty obvious that b - e is not less than a (because it's the LUB for the set), and so b - e/2 cannot be less than a.He did say it. He said "which contradicts the fact that c = sup{(a,b)} . This completes the proof. ".Also, it's pretty obvious that b - e is not less than a (because it's the LUB for the set), and so b - e/2 cannot be less than a.
  • #1
jgens
Gold Member
1,593
50

Homework Statement



Consider the open interval [itex](a,b)[/itex]. Prove that [itex]\mathrm{sup}{(a,b)} = b[/itex].

Homework Equations



N/A

The Attempt at a Solution



I'm terrible at these proofs so I would appreciate it if someone could verify (or correct) my solution.

Proof: Clearly [itex]b[/itex] is an upper bound since [itex]\forall{x} \in (a,b)[/itex] we have the strict inequality [itex]a < x < b[/itex]. Now, suppose that [itex]b[/itex] is not the least upper bound. Letting [itex]c = \mathrm{sup}{(a,b)}[/itex] there must be some real [itex]\varepsilon > 0[/itex] such that [itex]b - \varepsilon = c[/itex]. However, since [itex]\varepsilon > 0[/itex] this implies that [itex]\frac{\varepsilon}{2} > 0[/itex] and similarly that [itex]b > b - \frac{\varepsilon}{2} > b - \varepsilon = c[/itex] which contradicts the fact that [itex]c = \mathrm{sup}{(a,b)}[/itex]. This completes the proof.

I know that my proof is definitely wordy, but is it correct? Thanks for any suggestions!
 
Physics news on Phys.org
  • #2
jgens said:

Homework Statement



Consider the open interval [itex](a,b)[/itex]. Prove that [itex]\mathrm{sup}{(a,b)} = b[/itex].

Homework Equations



N/A

The Attempt at a Solution



I'm terrible at these proofs so I would appreciate it if someone could verify (or correct) my solution.

Proof: Clearly [itex]b[/itex] is an upper bound since [itex]\forall{x} \in (a,b)[/itex] we have the strict inequality [itex]a < x < b[/itex]. Now, suppose that [itex]b[/itex] is not the least upper bound. Letting [itex]c = \mathrm{sup}{(a,b)}[/itex] there must be some real [itex]\varepsilon > 0[/itex] such that [itex]b - \varepsilon = c[/itex]. However, since [itex]\varepsilon > 0[/itex] this implies that [itex]\frac{\varepsilon}{2} > 0[/itex] and similarly that [itex]b > b - \frac{\varepsilon}{2} > b - \varepsilon = c[/itex] which contradicts the fact that [itex]c = \mathrm{sup}{(a,b)}[/itex]. This completes the proof.

I know that my proof is definitely wordy, but is it correct? Thanks for any suggestions!

You may have the basics of a proof there, but it is definitely unclear. Why dose [itex]b > b - \frac{\varepsilon}{2} > b - \varepsilon = c[/itex] contradict the assumption that c is the supremum? If you clear that up, then the proof should be fine. Remember, there are 2 properties c should satisfy to be the supremum: It should be an upper bound, and every other upper bound should be greater than it. Which, if any, of these properties have you shown c not to satisfy?
 
  • #3
The proof seems fine to me.
 
  • #4
Yeah it looks clear to me. I mean the OP demonstrates that b - e/2 is in the open interval and (a,b) is greater than the supposed sup (a,b), so there's really not much else to say...
 
  • #5
snipez90 said:
Yeah it looks clear to me. I mean the OP demonstrates that b - e/2 is in the open interval and (a,b) is greater than the supposed sup (a,b), so there's really not much else to say...

But he doesn't. He doesn't explicitly say it and he doesn't restrict e to eliminate the possibility that b-e/2 < a so that it is not in the interval.
 
  • #6
LeonhardEuler said:
But he doesn't. He doesn't explicitly say it and he doesn't restrict e to eliminate the possibility that b-e/2 < a so that it is not in the interval.

He did say it. He said "which contradicts the fact that [tex] c = sup{(a,b)} [/tex] . This completes the proof. ".

Also, it's pretty obvious that b - e is not less than a (because it's the LUB for the set), and so b - e/2 cannot be less than a.

I see what you mean, but in my opinion, being that wordy would just make the proof longer than it has to be, without really contributing anything.
 

1. What is the definition of supremum?

The supremum of a set is the least upper bound, or the smallest number that is greater than or equal to all the numbers in the set.

2. How is supremum different from maximum?

Supremum and maximum both refer to the largest number in a set, but supremum can be equal to one of the numbers in the set, while maximum must be strictly greater than all the numbers in the set.

3. How do you prove that supremum of (a,b) is equal to b?

To prove that supremum of (a,b) is equal to b, we must show that b is the smallest number that is greater than or equal to all the numbers in the set (a,b). This can be done by showing that b is an upper bound for the set and that no number less than b is an upper bound for the set.

4. Can the supremum of a set be infinite?

Yes, the supremum of a set can be infinite. This occurs when the set does not have a largest number, such as in the set of all positive real numbers.

5. What is an example of a set where the supremum is not equal to any number in the set?

An example of a set where the supremum is not equal to any number in the set is the set of all real numbers between 0 and 1, where the supremum is equal to 1, but there is no number in the set that is equal to 1.

Similar threads

  • Calculus and Beyond Homework Help
Replies
2
Views
208
  • Calculus and Beyond Homework Help
Replies
12
Views
1K
  • Calculus and Beyond Homework Help
Replies
3
Views
5K
  • Calculus and Beyond Homework Help
Replies
7
Views
709
  • Math Proof Training and Practice
Replies
8
Views
826
  • Calculus and Beyond Homework Help
Replies
6
Views
2K
  • Calculus and Beyond Homework Help
Replies
3
Views
2K
  • Calculus and Beyond Homework Help
Replies
15
Views
1K
  • Calculus and Beyond Homework Help
Replies
7
Views
1K
  • Calculus and Beyond Homework Help
Replies
5
Views
2K
Back
Top